Download 1.Which of the following drugs is most likely responsible for this

Survey
yes no Was this document useful for you?
   Thank you for your participation!

* Your assessment is very important for improving the workof artificial intelligence, which forms the content of this project

Document related concepts
no text concepts found
Transcript
Anticancer &
Immunosupressants Q.
Questions 1-3. For each of the following statements, select the most
appropriate drug which is BEST characterized by the statement.
Answers may be used more than once.
(A)
Doxorubicin
(B)
Cyclophosphamide
(C)
Vincristine
(D)
Cisplatin
(E)
Methotrexate
1.Associated with the highest incidence of severe nausea during
chemotherapy
D
2.Binds to the catalytic site of Dihydrofolate Reductase (DHFR) and
interferes with pyrimidine synthesis
3.Causes cardiotoxicity
E
A
Questions 4-8. For each of the following statements, select
the most appropriate drug which is BEST characterized
by the statement. Answers may be used more than once.
(A)
Paclitaxel
(B)
Trastuzumab
(C)
Bleomycin
(D)
Finasteride
(E)
Tamoxifen
4. Usefulness in breast cancer therapy is due to
inhibition of HER2 protein
B
5. An antiandrogen often used to treat symptomatic
benign prostatic hyperplasia
D
6. Often results in pulmonary toxicity
C
7. Inhibits the depolymerization of microtubules
A
8. Isolated from the Pacific Yew tree
A
9.The following adverse effect is NOT
usually associated with cancer
chemotherapeutic agents such as the
alkylating agents and the antibiotics
(A) Alopecia
(B) Nausea
(C) Impotence
(D) Neutropenia
(E) Aglutition
10. Which one of the following chemotherapeutic
gents is specific for the M phase of the cell
cycle?
(A) Doxorubicin
(B) Busulfan
(C) Vincristine
(D) Topotecan
(E) Methotrexate
•
A 48-year-old woman is being treated for
breast carcinoma. Over the past few days, she
has been complaining of dysuria and
frequency. Laboratory examination reveals the
presence of microscopic hematuria. The next
day the patient develops gross hematuria.
Which of the following drugs could be used to
treat the side effect from the antineoplastic
medication taken by this patient?
(A) Cyclophosphamide
(B) Mitomycin
(C) Mesna
(D) Tamoxifen
(E) Vincristine
A 24-year-old man underwent treatment for Hodgkin
lymphoma 1 year ago. He presents with increasing
dyspnea and cough. Physical exam is remarkable for
rales bilaterally. Arterial blood gases show hypoxia,
and bilateral pulmonary infiltrates are seen on chest xray. Which of the following chemotherapeutic agents
most likely produced these side effects?
(A) Bleomycin
(B) Cyclophosphamide
(C) Doxorubicin
(D) Etoposide
(E) 5-Fluorouracil
(F) Streptozocin
(G) Vincristine
A 33-year-old man receiving chemotherapy for
testicular carcinoma develops signs of renal
tubular damage. Which of the following drugs
is most likely responsible for this
nephrotoxicity?
(A) Bleomycin
(B) Cisplatin
(C) Cyclophosphamide
(D) Vinblastine
(E) Vincristine
Questions 11-13. For each of the following statements, select the most
appropriate drug which is BEST characterized by the statement.
Answers may be used more than once.
(A)
(B)
(C)
(D)
(E)
Tacrolimus
Azathioprine
Infliximab
Palivizumab
Daclizumab
11. A monoclonal antibody useful in solid organ transplantation to
prevent HVGD
E
12. A human monoclonal antibody useful in the treatment of
Respiratory Syncytial Virus (RSV) infection in neonatal and
pediatric patients
D
13. Inhibits the action of FK binding protein and is useful as an
adjunct to Cyclosporine
A
Relevant Slide No-10(Chapter9)
• 1.An anticancer chemotherapeutic agent that acts by
first order kinetics would be expected to kill a
• a.Constant number of cancer cells
• b.Constant proportion of cancer cells
• c.Variable number of cells depending on the drug half life
• d.Variable number of cells depending on the proportion
of cells in S phase
• e.Variable proportion of cells depending on the number
of cells in Go phase
Relevant Slide No-15(Chapter9)
• 1.The tumor that is least susceptible to
cell-cycle-specific (CCS) anticancer
agents is
• a. Acute lymphoblastic leukemia
• b. Acute granulocytic leukemia
• c. Burkitt’s lymphoma
• d. Adenocarcinoma of the colon
• e. Choriocarcinoma
Relevant Slide No-15(Chapter9)
The tumor that is least susceptible to
cell-cycle-specific (CCS) anticancer
agents is
• a. Acute lymphoblastic leukemia
• b. Acute granulocytic leukemia
• c. Burkitt’s lymphoma
• d. Adenocarcinoma of the colon
• e. Choriocarcinoma
Relevant Slide No-16(Chapter9)
• Which of the following chemotherapeutic
agents is specific for the M phase of the cell
cycle?
•
•
•
•
A. Cytarabine
B. Daunorubicin
C. Hydroxyurea
D. Mechlorethamine
E. Vincristine
Relevant Slide No-16(Chapter9)
• 2.The phase of the cell cycle that is
resistant to most chemotherapeutic
agents and requires increased dosage
to obtain a response is the
a.M phase b. G2 phase c. S phase d. G0
phase e. G1 phase
Relevant Slide No-16(Chapter9)
Of the following, which is not a
CCS agent
• a. Mercaptopurine (6-MP)
• b. 5-FU
• c. Bleomycin
• d. Busulfan
• e. Vincristine
A 49-year-old man develops signs and
symptoms of peripheral neuropathy while
being treated for multiple myeloma with the M2 protocol consisting of carmustine,
cyclophosphamide, melphalan, prednisone
and vincristine. Which of these drugs is most
likely responsible for the peripheral
neuropathy in this patient?
A.
B.
C.
D.
E.
Carmustine
Cyclophosphamide
Melphalan
Prednisone
Vincristine
Answer: E
Vicristine neurotoxicity;
Vinblastine bone marrow toxicity
Relevant Slide No-18(Chapter9)
• 1.A 67 year old woman is being treated for
metastatic ovarian cancer with cisplatin and
cyclophosphamide. To prevent nausea and
vomiting, she is given an selective 5HT3
blocker. Which drug was she given?
• Dimenhydrinate
• Dronabinol
• Metoclopramide
• Ondansetron
• Prochlorperazine
Relevant Slide No-30(Chapter9)
• A nucleophilic attack on deoxyribonucleic
acid (DNA) that causes the disruption of base
pairing occurs as a result of the
administration of
• a. Cyclophosphamide
• b. 5-FU
• c. Methotrexate
• d. Prednisone
• e. Thioguanine
A 60-year-old man with metastatic
colorectal cancer is being treated with 5fluorouracil plus leucovorin and a drug that
is a potent inhibitor of topoisomerase I.
Which of the following is the most likely
topoisomerase I inhibitor used for the
treatment of this patient?
A.
B.
C.
D.
E.
Etoposide
Doxorubicin
Idarubicin
Irinotecan
Mitoxantrone
Answer: D
Irinotecan >> topoisomerase I;
All others >> topoisomerase II
Relevant Slide No-35(Chapter9)
• . A 45-year-old male has an insulinoma.
Which of the following agents is the
treatment of choice?
• a. Cyclophosphamide
• b. Carboplatin
• c. Vincristine
• d. Streptozocin
• e. Bleomycin
Relevant Slide No-37(Chapter9)
A patient is being treated for non-Hodgkin's lymphoma
with a nitrogen mustard-type antineoplastic agent
developed gross hematuria . Which of the following
agents could have been administered to prevent the
onset of the patient's symptoms?
A. Allopurinol
B. Leucovorin
C. C. Mesna
D. D. Penicillamine
E. E. Sodium thiosulfate
Relevant Slide No-38(Chapter9)
• 1.Which of the following drugs is most
likely responsible for this
nephrotoxicity?
• A. Bleomycin
• B. Cisplatin
• C. Cyclophosphamide
• D. Vinblastine
• E. Vincristine
Relevant Slide No-38(Chapter9)
• 2.A 45-year-old female treated for ovarian
cancer develops difficulty hearing. Which of
the following agents most likely caused
these findings?
• a. Paclitaxel
• b. Doxorubicin
• c. Bleomycin
• d. 5-FU
• e. Cisplatin
Relevant Slide No-41(Chapter9)
• Binding to the enzyme dihydrofolate
reductase is the mechanism of action
for
• a. Procarbazine
• b. Paclitaxel
• c. Methotrexate
• d. Ifosfamide
• e. Cladribine
Relevant Slide No-42(Chapter9)
• 1.A nine-year-old boy is diagnosed with acute
lymphoblastic leukemia. He is maintained on
methotrexate. A recent platelet count is below
normal, and a stool guaiac is 4+. Which of the
following agents should be administered to
counteract methotrexate toxicity?
• a. N-acetyl-L-cysteine
• b. Vitamin K
• c. Penicillamine
• d. Leucovorin
• e. Deferoxamine
Relevant Slide No-42(Chapter9)
• A 50-year-old female with rheumatoid
arthritis has developed erosions in her wrist
bones. Which of the following agents should
be administered? [Remember another
anticancer drug used in autoimmune
disease]
• a. Allopurinol
• b. Asparaginase
• c. Methotrexate
• d. Streptozocin
• e. 6-MP
Relevant Slide No-44(Chapter9)
• 1. A young adult patient with acute granulocytic
leukemia, treated with a combination of cytarabine
and thioguanine, is no longer responsive to the
therapy. The nonresponsiveness of the patient is
thought to be due to thioguanine. What is the
mechanism of resistance to thioguanine?
• a. Decreased uptake
• b. Increased efflux
• c. Increased alkaline phosphatase activity
• d. Increased production of trapping agents
• e. Increased DNA repair
Relevant Slide No-44(Chapter9)
• . Identify drug to administer as prechemotherapeutic agent to reduce
increased uric acid load caused by
chemotherapeutic agent.
• Answer: Allopurinol
Relevant Slide No-45(Chapter9)
• 1.A 60-year-old female treated for breast cancer
develops leukopenia and severe stomatitis and
oral ulcerations. Which of the following agents
most likely caused these findings?
• a. 5-FU
• b. Paclitaxel
• c. Cyclophosphamide
• d. Tamoxifen
• e. Carboplatin
Relevant Slide No-46(Chapter9)
• 1.which of the following agents would be
suitable for the treatment of patient's
lymphoma without further compromising his
immune system?
• A. Busulfan
• B. Cisplatin
• C. Cyclophosphamide
• D. Paclitaxel
• E. Vincristine
Relevant Slide No-46(Chapter9)
• Which of the following is considered to
be the effective mechanism of action of
the vinca alkaloids?
• a. Inhibition of the function of microtubules
• b. Damage and prevention of repair of
DNA
• c. Inhibition of DNA synthesis
• d. Inhibition of protein synthesis
• e. Inhibition of purine synthesis
Relevant Slide No-49(Chapter9)
• A 50-year-old female is treated with
paclitaxel. Of the following, how is
paclitaxel classified?
• a. An alkylating agent
• b. An antimetabolite
• c. A plant alkaloid
• d. An antibiotic
• e. A hormonal agent
Relevant Slide No-52(Chapter9)
• 1. A 58-year-old female develops a rather sudden
onset of orthopnea, paroxysmal nocturnal dyspnea,
and nocturia. On examination, the patient is
tachycardic and both pulmonary rales and a third
heart sound (S3) are noted. If the patient is receiving
antineoplastic therapy for treatment of breast
cancer, which of the following agents did she most
likely receive?
• A. Bleomycin
• B. Carmustine
• C. Cisplatin
• D. Doxorubicin
• E. Methotrexate
Relevant Slide No-52(Chapter9)
• . A 74-year-old woman with multiple myeloma is
being treated with high doses of doxorubicin
(Adriamycin). She has also received
cyclophosphamide and prednisone recently. During
his examination, the physician should check the
patient for
•
A. abdominal tenderness
•
B. bladder distention
•
C. limitation of movement
•
D. papilledema
•
E. pulmonary rales
Relevant Slide No-53(Chapter9)
• 1.A 32-year-old cancer patient, who has smoked two
packs of cigarettes a day for 10 years, presents a
decreased pulmonary function test. Physical
examination and chest x-rays suggest preexisting
pulmonary disease. Of the following drugs, which is
best not prescribed?
• a. Vinblastine
• b. Doxorubicin
• c. Mithramycin
• d. Bleomycin
• e. Cisplatin
Relevant Slide No-62(Chapter9)
• A 52-year-old woman presents to her physician for a checkup. She is recovering from a wrist fracture after a fall. Dual
energy x-ray absorptiometry of the hip had shown her to
have osteoporosis. She became menopausal at age 50 and
did not begin hormone replacement therapy because of a
strong family history of breast cancer. She now fears a
future hip fracture and would like to begin a bone loss
prevention regime. Which of the following pharmaceutical
agents is most appropriate for this patient?
•
A. Calcitonin nasal spray
•
B. Oral conjugated estrogen
•
C. Raloxifene
•
D. Tamoxifen
•
E. Transdermal estradiol
Relevant Slide No-63(Chapter9)
• . A 72-year-old man is diagnosed with
prostate cancer. He declines surgical
therapy, and is treated with flutamide. Which
of the following is the mechanism of action
of this drug?
• a.5 alpha-reductase inhibitor
• b.competitive antagonist at androgen
receptors
• c.GnRH analog
• d.LH agonist
• e.Testosterone agonist
Relevant Slide No-64(Chapter9)
• 1.A 16-year-old male treated for acute
lymphocytic leukemia develops severe
lumbar and abdominal pain. His serum
amylase is markedly elevated. Which of the
following agents most likely caused these
findings?
• a. 6-MP
• b. Asparaginase
• c. Doxorubicin
• d. Methotrexate
• e. Vincristine
Relevant Slide No-72(Chapter9)
• 1.A 58-year-old female develops a rather sudden
onset of orthopnea, paroxysmal nocturnal dyspnea,
and nocturia. On examination, the patient is
tachycardic and both pulmonary rales and a third
heart sound (S3) are noted. If the patient is receiving
antineoplastic therapy for treatment of breast
cancer, which of the following agents did she most
likely receive?
•
A. Bleomycin
•
B. Carmustine
•
C. Cisplatin
•
D. Doxorubicin
•
E. Methotrexate
Relevant Slide No-79(Chapter9)
• 1.A 30-year-old male with a two-year history of
chronic renal failure requiring dialysis consents to
transplantation. A donor kidney becomes available.
He is given cyclosporine to prevent transplant
rejection just before surgery. What is the most likely
adverse effect of this drug?
• a. Bone marrow depression
• b. Nephrotoxicity
• c. Oral and GI ulceration
• d. Pancreatitis
• e. Seizures
Relevant Slide No-79(Chapter9)
• The most effective drug for
immunosuppression of rejection of the
allografted kidney is
a. Azathioprine
• b. Cyclosporine
• c. 5-fluorouracil (5-FU)
• d. Cyclophosphamide
• e. Vincristine
Relevant Slide No-79(Chapter9)
• A 40-year-old female post–renal transplant
has developed evidence of osteoporosis,
most likely due to cyclosporine. Which of the
following agents might replace
cyclosporine?
• a. Allopurinol
• b. Asparaginase
• c. Methotrexate
• d. Streptozocin
• e. 6-MP
• f. Azathioprine
Relevant Slide No-79(Chapter9)
• . A 45-year-old female has a bone marrow transplant
for treatment of ovarian cancer. Cyclosporine is
given as an immunosuppressant. What is the
mechanism of action of cyclosporine?
• a. Direct destruction of proliferating lymphoid cells
• b. Inhibition of T cell response to cytokines
• c. Inhibition of folic acid metabolism
• d. Inhibition of factors that stimulate T cell growth
• e. Inhibition of enzymes that are related to purine
metabolism
Relevant Slide No-81(Chapter9)
• 1.A 25-year-old female post–renal transplant
shows signs of acute renal allograph
rejection. Of the following agents, which
should be administered?
• a. Interferon a
• b. Aldesleukin
• c. Muromonab-CD3
• d. Sargramostim
• e. Filgrastim